Digamos que apuntamos un láser que se proyecta en un $90^{\circ}$ esquina de un espejo formado por paredes perpendiculares.
Sé que no se puede utilizar la tangente ya que una derivada del límite del espejo no existiría en una esquina. Pero esto no me ha convencido de que no pueda existir un reflejo.
Así que me imaginé proyectando un rayo incidente de la misma pendiente lo más cerca posible de la esquina.
Pensé que el rayo de incidencia estaría en el mismo que el rayo de incidencia excepto que el rayo reflejado se mueve en la dirección opuesta.
Sin embargo, no estoy seguro de que esto sea correcto, ya que no tengo un espejo de esquina. Tampoco estoy seguro de que pueda existir un reflejo en una esquina.
¿Estoy en lo cierto o las leyes de la física dicen lo contrario? Y si no estoy en lo cierto, ¿dónde se proyectaría el rayo relfexionado?
3 votos
Esta es una situación típica en la que la aproximación del rayo falla y habría que utilizar las ecuaciones de onda para calcular lo que ocurre.
0 votos
Tengo pocos conocimientos de física, ya que no sé por dónde empezar. Tengo suficiente conocimiento de las matemáticas puras, así que tomé este enfoque. Voy a lo que puedo aprender sobre las ecuaciones de onda.
2 votos
Desgraciadamente no hay una teoría fácil para esto. Se pueden consultar documentos como math.wisc.edu/~jin/PS/corner.pdf y esta tesis para obtener algunas ideas e indicaciones sobre dónde buscar más.
1 votos
@CuriousOne Gracias. Me sorprende que un problema como este sea muy complicado.
0 votos
@CuriousOne En el papel, la reflexión de la luz se estima en una esquina hacia afuera y no hacia adentro. La esquina hacia adentro puede no tener los mismos principios.
1 votos
Los principios son los mismos, el resultado puede diferir un poco (pero puede haber una transformación geométrica que te lleve de un caso a otro, no estoy seguro de ello). Básicamente hay que seguir los métodos matemáticos desarrollados en estos artículos (también hay que seguir las citas) para derivar la dispersión efectiva causada por un borde afilado.
0 votos
@CuriousOne No creo que la aproximación del rayo se rompa necesariamente aquí. Falla para tu ejemplo porque la dispersión predicha cambia discontinuamente con la posición del haz de luz incidente. Pero eso no es cierto en este ejemplo.
0 votos
@CuriousOne Estoy de acuerdo en que genéricamente hay que ir a las ecuaciones de onda, eso sí.
0 votos
@knzhou: Aunque hay que describir la dispersión en el borde. Es una cuestión interesante si se puede utilizar la solución de la aproximación del rayo y luego superponer la dispersión del borde. Parece razonable creer que esto puede ser una buena aproximación, tal vez incluso una solución correcta, pero no sé lo suficiente sobre estos problemas para hacer esa afirmación.
0 votos
@CuriousOne : Tus comentarios realmente pertenecen a una Respuesta ( meta.physics.stackexchange.com/questions/8821/ ). Entonces el OP puede indicar si está satisfecho con su explicación. De lo contrario, parece que la pregunta no tiene una respuesta aceptada.